Not x = y = z x = y = z

Algebra Level 5

x 3 + y 3 + z 3 ( x + y + z ) ( x 2 + y 2 + z 2 ) \large\dfrac{x^3+y^3+z^3}{(x+y+z)(x^2+y^2+z^2)}

Let x , y , z x, y, z be non-negative real numbers such that x 2 + y 2 + z 2 = 2 ( x y + y z + z x ) x^2+y^2+z^2=2(xy+yz+zx) . Find the maximum value of the expression above.

Give your answer to 3 decimal places.


The answer is 0.611.

This section requires Javascript.
You are seeing this because something didn't load right. We suggest you, (a) try refreshing the page, (b) enabling javascript if it is disabled on your browser and, finally, (c) loading the non-javascript version of this page . We're sorry about the hassle.

3 solutions

From the hypothesis, we get x y + y z + z x = 1 4 ( x + y + z ) 2 xy+yz+zx=\dfrac{1}{4}(x+y+z)^2 .

So, P = x 3 + y 3 + z 3 2 ( x + y + z ) ( x y + y z + z x ) = 2 ( x 3 + y 3 + z 3 ) ( x + y + z ) 3 P=\dfrac{x^3+y^3+z^3}{2(x+y+z)(xy+yz+zx)}=\dfrac{2(x^3+y^3+z^3)}{(x+y+z)^3}

= 2 [ ( x x + y + z ) 3 + ( y x + y + z ) 3 + ( z x + y + z ) 3 ] \qquad \;\;=2\left[\left(\dfrac{x}{x+y+z}\right)^3+\left(\dfrac{y}{x+y+z}\right)^3+\left(\dfrac{z}{x+y+z}\right)^3\right]

Let a = x x + y + z ; b = y x + y + z ; c = z x + y + z a=\dfrac{x}{x+y+z}; b=\dfrac{y}{x+y+z}; c=\dfrac{z}{x+y+z} , we have { a + b + c = 1 a b + b c + c a = 1 4 \left\{\begin{array}{l}a+b+c=1\\ab+bc+ca=\dfrac{1}{4}\end{array}\right.

Or { b + c = 1 a b c = a 2 a + 1 4 \left\{\begin{array}{l}b+c=1-a\\bc=a^2-a+\dfrac{1}{4}\end{array}\right.

From the inequality ( b + c ) 2 4 b c (b+c)^2\ge 4bc , we get 0 a 2 3 0\le a\le\dfrac{2}{3} .

We have:

P = 2 ( a 3 + b 3 + c 3 ) P=2(a^3+b^3+c^3)

= 2 ( a 3 + ( b + c ) 3 3 b c ( b + c ) \quad =2(a^3+(b+c)^3-3bc(b+c)

= 2 ( a 3 + ( 1 a ) 3 3 ( a 2 a + 1 4 ) ( 1 a ) ) \quad =2\left(a^3+(1-a)^3-3\left(a^2-a+\dfrac{1}{4}\right)(1-a)\right)

= 6 a 3 6 a 2 + 3 2 a + 1 2 \quad =6a^3-6a^2+\dfrac{3}{2}a+\dfrac{1}{2}

= 11 18 + 1 18 ( 3 a 2 ) ( 6 a 1 ) 2 11 18 \quad =\dfrac{11}{18}+\dfrac{1}{18}(3a-2)(6a-1)^2\le\dfrac{11}{18}

So, max P = 11 18 0.611 \max P=\dfrac{11}{18}\boxed{\approx 0.611} .

The equality holds when ( x , y , z ) = ( 4 k , k , k ) (x,y,z)=(4k,k,k) or any permutations.

Nice solution! :)

Curtis Clement - 5 years, 8 months ago

x x , y y and z z should be positive real number instead of non-negative. The expression is maximum when x = y = z = 0 x=y=z=0 , which satisfies x 2 + y 2 + z 2 ) = 2 ( x y + y z + z x ) x^2+y^2+z^2) = 2(xy+yz+zx)

Chew-Seong Cheong - 5 years, 6 months ago

Log in to reply

If x=y=z=0, P = 0/0 = undefined

Pi Han Goh - 5 years, 6 months ago

Log in to reply

Yes, I realized it later.

Chew-Seong Cheong - 5 years, 6 months ago

I could have got the same answer. I did some calculation mistake.

Srikanth Tupurani - 2 years, 1 month ago

Similar expession I got but instead 11/18 I got 12/18. Some serious mistake happened. Nice solution.

Srikanth Tupurani - 2 years, 1 month ago

Let's use the following notation: S 1 = x + y + z S_1=x+y+z , S 2 = x y + x z + y z S_2=xy+xz+yz and S 3 = x y z S_3=xyz . Then, from the hypothesis we know that S 1 2 = 4 S 2 S_1^2=4S2 , and the expression we want to maximize is:

P = x 3 + y 3 + z 3 ( x + y + z ) ( x 2 + y 2 + z 2 ) = S 1 3 3 S 1 S 2 + 3 S 3 2 S 1 S 2 P=\dfrac{x^3+y^3+z^3}{(x+y+z)(x^2+y^2+z^2)}=\dfrac{S_1^3-3S_1S_2+3S3}{2S_1S_2}

Then, simplify it using the hypothesis, in a way such that we only need S 1 S_1 and S 3 S_3 :

P = 1 2 + 6 S 3 S 1 3 P=\dfrac{1}{2}+\dfrac{6S_3}{S_1^3}

Now, consider a polynomial with roots x , y , z x,y,z , of course it is P ( x ) = x 3 S 1 x 2 + S 2 x S 3 P(x)=x^3-S_1x^2+S_2x-S_3 . Then, for the roots to be real, the discriminant of this polynomial must be 0 \geq 0 , so:

S 1 2 S 2 2 4 S 2 3 4 S 1 3 S 3 + 18 S 1 S 2 S 3 27 S 3 2 0 S_1^2S_2^2-4S_2^3-4S_1^3S_3+18S_1S_2S_3-27S_3^2 \geq 0

Again, using the hypothesis, we get:

1 2 S 1 3 27 S 3 0 \dfrac{1}{2}S_1^3-27S_3 \geq 0

Hence, S 3 S 1 3 1 54 \dfrac{S_3}{S_1^3} \leq \dfrac{1}{54}

Finally, P 1 2 + 6 ( 1 54 ) = 11 18 P \leq \dfrac{1}{2}+6\left(\dfrac{1}{54}\right)=\boxed{\dfrac{11}{18}}

I did something similar. Considering the cubic. And tried to find the maximum possible value for the product of roots.

Srikanth Tupurani - 2 years, 1 month ago
Joel Tan
Dec 31, 2015

Let x + y + z = 2 S x+y+z=2S for some S S .

From what is given ( x + y + z ) 2 = 4 ( x y + y z + z x ) (x+y+z)^{2}=4(xy+yz+zx) hence x y + y z + z x = S 2 , x 2 + y 2 + z 2 = 2 S 2 xy+yz+zx=S^{2}, x^{2}+y^{2}+z^{2}=2S^{2} . Thus the denominator is ( x + y + z ) ( x 2 + y 2 + z 2 ) = 4 S 3 (x+y+z)(x^{2}+y^{2}+z^{2})=4S^{3} .

Also ( x + y z ) 2 = 4 x y (x+y-z)^{2}=4xy .

Consider the numerator.

x 3 + y 3 + z 3 = ( x + y + z ) ( x 2 + y 2 + z 2 x y y z z x ) + 3 x y z = 2 S 3 + 3 x y z 2 S 3 + 4 9 S 3 x^{3}+y^{3}+z^{3}=(x+y+z)(x^{2}+y^{2}+z^{2}-xy-yz-zx)+3xyz=2S^{3}+3xyz \leq 2S^{3}+\frac{4}{9}S^{3} .

The last inequality follows because

16 x y z = 4 z ( 4 x y ) = 4 z ( x + y z ) ( x + y z ) ( 2 ( x + y + z ) 3 ) 3 = 64 27 S 3 16xyz=4z(4xy)=4z(x+y-z)(x+y-z) \leq (\frac{2(x+y+z)}{3})^{3}=\frac{64}{27}S^{3} by AM-GM inequality. (We can make sure all terms in the AM-GM are positive by letting x y z x \geq y \geq z without loss of generality).

Hence x 3 + y 3 + z 3 ( x + y + z ) ( x 2 + y 2 + z 2 ) 2 S 3 + 4 9 S 3 4 S 3 = 11 18 \frac{x^{3}+y^{3}+z^{3}}{(x+y+z)(x^{2}+y^{2}+z^{2})} \leq \frac{2S^{3}+\frac{4}{9}S^{3}}{4S^{3}}=\frac{11}{18} .

However when x = 4 y = 4 z x=4y=4z we have equality, so answer is 11 18 \frac{11}{18} .

0 pending reports

×

Problem Loading...

Note Loading...

Set Loading...